Frage:
Wie erzeugen bewegte Ladungen Magnetfelder?
claws
2013-05-21 15:00:07 UTC
view on stackexchange narkive permalink

Ich unterrichte Schüler. Ich habe ihnen immer Folgendes beigebracht:

Ein geladenes Teilchen, das sich ohne Beschleunigung bewegt, erzeugt sowohl ein elektrisches als auch ein magnetisches Feld .

Es erzeugt ein elektrisches Feld, weil es ein Ladungsteilchen ist. Aber wenn es in Ruhe ist, erzeugt es kein Magnetfeld. Wenn es sich zu bewegen beginnt, erzeugt es plötzlich ein Magnetfeld. Warum? Was passiert damit, wenn es sich in Bewegung setzt? Was erzeugt ein Magnetfeld, wenn es sich zu bewegen beginnt?

Dem Teilchen passiert nichts, wodurch es ein Magnetfeld erzeugt, wenn es sich zu bewegen beginnt: Elektrisches und magnetisches Feld sind Bestandteile des elektromagnetischen Feldes, das eine Einheit bildet, ähnlich wie Energie und Impuls Bestandteile des 4-Impulses sind. Im Ruhezustand eines geladenen Teilchens verschwinden die magnetischen Komponenten ebenso wie sein 3-Impuls, und nur die zeitlichen (das elektrische Feld bzw. die Energie) bleiben übrig
@Christoph: Sie haben viele neue Wörter verwendet, die ich nicht verstehe. Ich studiere kein Hauptfach Physik. Könnten Sie etwas (einfaches) zum Lesen vorschlagen?
Sie müssen sich über die spezielle Relativitätstheorie informieren. Wenn Sie etwas warten, werde ich meinen Kommentar zu einer richtigen Antwort erweitern ...
http://physics.stackexchange.com/q/54942/ überprüfen Sie es.
Verwandte: http://physics.stackexchange.com/q/3618/
Verwandte Veritasium / Minutephysics-Video: http://www.youtube.com/watch?v=1TKSfAkWWN0
Ich empfehle die Arbeit von Professor Oleg D. Jefimenko mit dem Titel: "Kausalität, elektromagnetische Induktion und Gravitation: Ein anderer Ansatz zur Theorie elektromagnetischer Felder und Gravitationsfelder, 2. Aufl."In dieser Arbeit löst der gute Professor Maxwells Gleichungen und zeigt, dass elektrischer Strom und Magnetfelder keine kausalen Quellen voneinander sind, sondern eine doppelte Einheit, die gleichzeitig von einer gemeinsamen Quelle verursacht wird.
Ein sich bewegender Magnet hat auch ein elektrisches Feld.Wenn Sie ein geladenes Teilchen oder einen beschleunigten Magneten beschleunigen, erzeugen Sie eine elektromagnetische Welle, da die beiden Felder eine Manifestation derselben Kraft sind.
Warum ohne Beschleunigung?Wenn es sich mit Beschleunigung bewegt, was passiert dann?
Sieben antworten:
user10851
2013-05-22 00:31:32 UTC
view on stackexchange narkive permalink

Wenn Sie mit der speziellen Relativitätstheorie nicht gut vertraut sind, gibt es keine Möglichkeit, dieses Phänomen wirklich zu erklären . Das Beste, was Sie tun können, ist, Ihnen Regeln zu geben, die von esoterischen Ideen wie "elektromagnetisches Feld" und "Lorentz-Invarianz" durchdrungen sind. Natürlich ist dies nicht das, wonach Sie suchen, und das zu Recht, denn in der Physik sollte es niemals darum gehen, Regeln zu akzeptieren, die ohne Begründung von oben überliefert wurden.

Tatsache ist, Magnetismus ist nichts anderes als Elektrostatik kombiniert mit spezieller Relativitätstheorie . Leider werden Sie nicht viele Bücher finden, die dies erklären - entweder glauben die Autoren fälschlicherweise, dass Maxwells Gleichungen keine Rechtfertigung haben und im Glauben akzeptiert werden müssen, oder sie sind zu sehr in ihrer eigenen esoterischen Notation verstrickt, um innezuhalten, um zu überlegen, was sie sagen. Das einzige Buch, von dem ich weiß, dass es das Thema richtig behandelt, ist Purcells Elektrizität und Magnetismus , das kürzlich in einer dritten Ausgabe erneut veröffentlicht wurde. (Die zweite Ausgabe funktioniert einwandfrei, wenn Sie eine Kopie finden.)

Eine kurze, heuristische Gliederung der Idee lautet wie folgt. Angenommen, es gibt eine Linie positiver Ladungen, die sich entlang der $ z $ -Achse in die positive Richtung bewegen - einen Strom. Stellen Sie sich eine positive Ladung $ q $ vor, die sich bei $ (x, y, z) = (1,0,0) $ befindet und sich in der negativen $ z $ -Richtung bewegt. Wir können sehen, dass aufgrund all dieser Ladungen eine gewisse elektrostatische Kraft auf $ q $ ausgeübt wird.

Aber versuchen wir etwas Verrücktes - lassen Sie uns in den Bezugsrahmen von $ q $ schlüpfen. Schließlich sollten die Gesetze der Physik für alle Gesichtspunkte gelten. Es ist klar, dass sich die Ladungen, aus denen der Strom besteht, in diesem Rahmen schneller bewegen werden. Aber das macht nicht viel, denn schließlich kümmert sich die Coulomb-Truppe eindeutig nicht um die Geschwindigkeit der Ladungen, sondern nur um ihre Trennung. Aber die spezielle Relativitätstheorie sagt uns etwas anderes. Es heißt, dass die aktuellen Gebühren näher beieinander liegen werden. Wenn sie im ursprünglichen Frame durch Intervalle $ \ Delta z $ voneinander beabstandet waren, haben sie in diesem neuen Frame einen Abstand $ \ Delta z \ sqrt {1-v ^ 2 / c ^ 2} $, wobei $ v $ ist die Geschwindigkeit von $ q $ im Originalrahmen. Dies ist die berühmte Längenkontraktion, die durch spezielle Relativitätstheorie vorhergesagt wird.

Wenn die aktuellen Ladungen näher beieinander liegen, fühlt sich $ q $ eindeutig elektrostatisch an Kraft aus der $ z $ -Achse als Ganzes. Es wird eine zusätzliche Kraft in der positiven $ x $ -Richtung von der Achse weg erfahren, die über das hinausgeht, was wir vorhergesagt hätten, wenn wir nur im Laborrahmen gesessen hätten. Grundsätzlich ist das Coulombsche Gesetz das einzige Kraftgesetz, das auf eine Ladung wirkt, aber nur der Restrahmen der Ladung gilt für die Verwendung dieses Gesetzes, um zu bestimmen, welche Kraft die Ladung empfindet.

Anstatt Wir wandeln uns ständig zwischen Frames hin und her und erfinden das Magnetfeld als mathematisches Gerät, das dasselbe erreicht. Wenn es richtig definiert ist, wird es diese anomale Kraft, die scheinbar von der Ladung erfahren wird, vollständig erklären, wenn wir sie nicht in ihrem eigenen Ruhezustand beobachten. In dem Beispiel, das ich gerade durchlaufen habe, sagt Ihnen die rechte Regel, dass wir dem Strom, der um die $ z $ -Achse kreist, ein Magnetfeld zuschreiben sollten, so dass er in die positive $ y $ -Richtung auf die Position von $ zeigt q $. Die Geschwindigkeit der Ladung ist in der negativen $ z $ -Richtung, und so zeigt $ q \ vec {v} \ times \ vec {B} $ in die positive $ x $ -Richtung, genau wie wir aus dem Ändern von Referenzrahmen gelernt haben .

Vielen Dank, dass Sie mir gesagt haben, was ich nicht weiß, anstatt meine Frage direkt zu beantworten. Vielen Dank für die Buchempfehlung. Vielen Dank, dass Sie eine so einfache Sprache verwenden, um mir neue Dinge zu erklären. Liebte deine Antwort :)
Beeindruckend! (+1) Eine der legendären verdammt guten Antworten auf eine scheinbar einfache Frage, die wir hier von Zeit zu Zeit sehen. Es erinnert mich an die kinetische Energie und die Tonstücke von R.M.
Ich denke nicht, dass das die ganze Geschichte ist. Das elektromagnetische Feld kann nur bei $ P <0 $ auf ein elektrisches reduziert werden. Im Fall von $ P> 0 $ bricht Ihre Beschreibung zusammen und Sie wären gezwungen, die Magnetostatik als die grundlegende Interaktion zu betrachten, die auf verschiedene Frames übertragen wird ...
@Christoph Ich versuche nicht, das Magnetfeld loszuwerden, da dies die Umwandlung der Ladungen, die es erzeugen, in den Restrahmen bedeuten würde, und ein solcher globaler Rahmen wird für die meisten aktuellen Verteilungen eindeutig nicht existieren. Ich verwandle mich in den Rahmen der * Testladung *, der immer durchgeführt werden kann und in dem es keinen * Effekt * des Magnetismus auf die Ladung gibt.
@ChrisWhite: danke für die Klarstellung, ich habe die Prämisse Ihres Setups nicht verstanden; Ein nicht verschwindendes Magnetfeld * wird * im Allgemeinen jedoch einen Einfluss auf die Testladung haben (Ausrichtung des intrinsischen magnetischen Moments). Kann das auch sinnvoll modelliert werden, z. B. indem der Rest der Testladung gedreht wird?
@Christoph Intrinsic Spin - das ist eine Komplikation, die ich zugeben muss, nicht durchdacht zu haben.
Netter Versuch, aber Sie hätten wirklich Purcells Argument kopieren sollen, in dem er einen Elektronenstrom betrachtet, der sich in einem Draht bewegt, in dem sich die elektrischen Felder im Laborrahmen gegenseitig aufheben, aber nicht im Elektronenrahmen.
@ChrisWhite Ich hatte eine Frage zu Ihrer Antwort: Sollte der Abstand zwischen den aktuellen Ladungen nicht konstant bleiben? Ich frage das, weil ich nach dem Paradoxon des Bell-Raumschiffs den Eindruck habe, dass ein stationärer Beobachter, der zwei durch ein Leerzeichen 'x' getrennte Körper auf eine Geschwindigkeit 'v' beschleunigen sieht, keinen Unterschied in der Lücke beobachten würde zwischen den Körpern, obwohl sie sich zusammenziehen würden.
@user1218748 Ich habe noch nie von diesem Paradoxon gehört. Es scheint von den Feinheiten der Implementierung der Beschleunigung abzuhängen (insbesondere wenn sie gestoppt wird). Das Beschleunigen von Frames ist schwierig, insbesondere wenn versucht wird, sie über große Entfernungen zu verlängern. Natürlich müssen Sie vorsichtig sein, denn Ihre Interpretation würde bedeuten, dass eine Längenkontraktion überhaupt nicht stattfinden kann - betrachten Sie einfach alle Objekte als Sammlungen ihrer Atome. In jedem Fall klingt dies als Grundlage für eine separate Frage.
@ChrisWhite: Angenommen, Ihre Ladung nähert sich dem Strom, anstatt sich parallel dazu zu bewegen. Was sagen Sie dann voraus? E & M bietet mehr als Coulomb + SR. Bitte überprüfen Sie Jackson 12.2.
Zu Ihrer Information, Griffiths gibt dies auch in seinem Buch eine schöne Behandlung.
Vielleicht nehme ich Ihre Worte zu wörtlich, aber ich denke, dass der Satz „Magnetismus ist nichts anderes als Elektrostatik kombiniert mit SR“ falsch ist.Das einfachste Beispiel, das Sie nicht mit Elektrostatik behandeln können, ist das einer beschleunigenden Ladung.Wie Feynman in Vorlesung 26 lehrt, können Sie die gesamten Felder aus dem Couloumbschen Gesetz erhalten, wenn Sie die Ideen hinzufügen, dass (1) das Potential $ (\ rho, A_x, A_y, A_z) $ ein Tetra-Vektor ist und (2) dass eshängt nur von Positionen und Geschwindigkeiten zum verzögerten Zeitpunkt ab.
Gibt es ein Analogon für die anderen Kräfte, zB: Schwerkraft?Ich sehe keinen Grund, warum es keinen geben sollte, aber gleichzeitig ist mir so etwas nicht bewusst.
Ich fürchte, diese Antwort ist falsch.Es wiederholt einen Mythos.Sie können keine magnetische Rotationsbewegung aus der Längenkontraktion in einem linearen Draht ableiten.Siehe [diese Frage] (http://physics.stackexchange.com/questions/210477/how-can-length-contraction-result-in-electron-circular-motion-in-a-magnetic-fiel) und die zugehörige Diskussion.
@JohnDuffield Bei der Frage, die Sie verlinkt haben, scheint mir die am besten gewählte Antwort mit dieser Antwort übereinzustimmen, oder habe ich etwas falsch verstanden?
180 Stimmen, aber es ist falsch und ich bitte Sie, Ihre Antwort einer größeren Änderung zu unterziehen.Die Frage, ob Magnetismus vollständig aus Elektrostatik plus Relativitätstheorie abgeleitet werden kann, wurde lange in Betracht gezogen und kann es sicherlich nicht.Ein Grund ist, dass es Ihnen nicht die Auswirkungen der Beschleunigung sagt.Außerdem trägt $ \ bf B $ nicht nur eine Kraft auf ein sich bewegendes Teilchen bei, sondern wirkt auch als Quelle für $ \ bf E $ durch Induktion.Wie erklären Sie eine elektromagnetische Welle aus der Elektrostatik?Das geht nicht.Schließlich gibt es keinen Rahmen, in dem $ \ bf B $ verschwindet, wenn $ {\ bf E} \ cdot {\ bf B} \ ne 0 $.
Ich bin Laie, aber diese Antwort stört mich aus zwei Gründen: - Die Driftgeschwindigkeit eines Elektrons durch einen Kupferdraht mit einer Querschnittsfläche von 3,00 x 10-6 m² und einem Strom von 10 A beträgt ungefähr 2,5 x 10-4 m / s, eine sehr niedrige relativistische Geschwindigkeit.Das elektrische Feld wird in der Lichtgeschwindigkeit hart sein. - Sie können auch ein elektrisches Feld erzeugen, das einen Magneten bewegt.Sie können sogar eine elektromagnetische Welle erzeugen, mit der Sie den Magneten beschleunigen, genau wie ein elektrisches Teilchen. Werden diese Fragen nicht besser mit der Quantenmechanik und ihrer Feldtheorie beantwortet?
Christoph
2013-05-21 16:04:01 UTC
view on stackexchange narkive permalink

Elektrische und magnetische Felder sind das Aussehen des elektromagnetischen Feldes ' ' aus einem bestimmten (Trägheits-) Referenzrahmen.

Nehmen Sie eine Ladung Teilchen: In seinem Ruhezustand scheint es nur ein elektrisches Feld und überhaupt kein magnetisches Feld zu erzeugen. Aus einem anderen Bezugsrahmen (insbesondere einem in Bezug auf die Relativbewegung) sehen wir, wie sich die Ladung bewegt, also ein Strom, der ebenfalls ein Magnetfeld erzeugt.

Dies bedeutet nicht, dass das Teilchen eingesetzt wird Die Bewegung hat irgendwie einen Schalter innerhalb des Partikels umgelegt - vielmehr ist es ein Artefakt unserer Wahl des Referenzrahmens: Beobachter in Relativbewegung messen unterschiedliche Stärken elektrischer und magnetischer Felder auf dieselbe Weise, wie sie unterschiedliche Geschwindigkeiten und Impulse messen.

Nehmen wir ein Feld ungleich Null mit $ P, Q = 0 $, dh $ \ mathbf E ^ 2 = \ mathbf B ^ 2 $ und $ \ mathbf E \ perp \ mathbf B \ ;. $ Ein Beispiel wäre eine ebene elektromagnetische Welle, die für alle wie eine ebene Welle aussieht.

Nun sei $ P. \ not = 0 $ aber $ Q = 0 \ ;. $ Dann können wir fra finden Referenznummern, bei denen entweder das elektrische (im Fall von $ P>0 $) oder das magnetische Feld (im Fall von $ P<0 $) verschwindet. Der Rest unseres geladenen Teilchens wäre ein solcher.

Für weitere Details müssen Sie in der Literatur zur speziellen Relativitätstheorie nachsehen.

Bedeutet das also, dass ein geladenes Teilchen kein elektrisches Feld in Bewegung erzeugt?
Francisco Muller
2014-03-07 07:49:11 UTC
view on stackexchange narkive permalink

Obwohl Chris Whites Antwort auf die Frage "Warum bewegliche Ladungen ein Magnetfeld erzeugen?" Gepostet von einem High School Lehrer (Claws) im letzten Jahr, wurde als beste Antwort ausgewählt, ich denke, es enthält mehrere Fallstricke. Chris White stellt sich einen Strom positiver Ladungen vor, der in Richtung der Achse $ + z $ fließt, während sich eine Testladung $ + q $, die sich ursprünglich bei $ (1,0,0) $ befindet, in die entgegengesetzte Richtung $ (- z) $ bewegt mit Geschwindigkeit $ v $. Als nächstes will er beweisen, dass der Beobachter, wenn er sich im Rahmen der sich bewegenden Testladung befindet, zusätzlich zu der regulären elektrostatischen Coulomb-Kraft (Abstoßungskraft), die auf die Testladung wirkt, eine zusätzliche Abstoßung im $ + x $ sieht Richtung, deren Ursprung völlig relativistisch ist. Dies geschieht, sagt er, weil die ursprüngliche Trennung $ Δz_0 $ zwischen den Ladungen (vom Lab-Ruhebild aus gesehen) jetzt auf $ Δz = Δz_0 \ sqrt {(1-v ^ 2 / c ^ 2)} $ ( Die "berühmte" Lorentz-Kontraktion).

Folglich werden alle Abstände der fließenden Ladungen zur Testladung kleiner (als ob die Ladungsdichte zunehmen würde) und daher nehmen auch die Coulomb-Abstoßungen zu. Dieser Überschuss an Abstoßung ist die „illusorische“ Magnetkraft, die der Laborbeobachter sieht, wenn sich die Testladung mit der Geschwindigkeit $ v $ in Richtung $ –z $ bewegt.

Kurz gesagt: Es gibt keine intrinsische Magnetkraft . Alles ist die Coulomb-Kraft, gesehen vom Lab-Rahmen (reine elektrostatische Kraft) oder gesehen vom beweglichen Ladungsrahmen (elektrostatisch plus mehr Coulomb-Abstoßung). Wir können hier alle quantitativen Details umgehen, die Weiß ebenfalls auslässt, aber wir können die Fallstricke nicht übersehen :

  1. Zunächst gibt es einen verbalen Widerspruch: Um das kontrahierte $ Δz $ zu bemerken, das kleiner als $ Δz_0 $ ist, muss sich der Beobachter mit der Ladung $ q $ in Ruhe befinden (d. H. Sich mit der Ladung bewegen). Aber am Ende sagt White, dass die neue „anomale Kraft, die scheinbar von der Ladung erfahren wird“ (d. H. Das definierte Magnetfeld), auftritt, „wenn wir sie nicht in ihrem eigenen Ruhezustand beobachten“ (Hervorhebung von mir). Also, was ist der Deal? Um die zusätzliche Coulomb-Kraft (magnetisch) vorherzusagen, müssen wir den Rahmen der sich bewegenden Ladung übernehmen. Aber um es zu beobachten, müssen wir im Lab-Rahmen bleiben, der NICHT der sich bewegende Ladungsrahmen ist.
  2. In der gleichen Weise gibt es eine numerische Falle: die neue (kontrahierte) Ladungstrennung Δz, die vom Rahmen beobachtet wird der sich bewegenden Ladung wird berechnet als $ Δz = Δz_0 \ sqrt {(1-v ^ 2 / c ^ 2)} $ wobei $ v $, sagt Weiß, "die Geschwindigkeit von $ q $ im ursprünglichen Rahmen" ist. Er hätte nicht $ v $, sondern $ 2v $ setzen sollen, da die Relativgeschwindigkeit zwischen dem steigenden Ladungsstrom $ v $ und der fallenden Testladung $ -v $ $ v - (- v) = 2v ist $ .So sollte der Kontraktionsfaktor $ \ sqrt {1-4v ^ 2 / c ^ 2} $ sein.
  3. Wenn wir außerdem die von Weiß verwendete heuristische Strategie verwenden, erreichen wir einen Widerspruch: Beginnen Sie mit Alle Gebühren in Ruhe: die $ z $ -Achse voller Gebühren und die Testgebühr bei $ (1,0,0) $. Nennen Sie $ Δz_0 $ die Trennung zwischen allen Ladungen in Ruhe. Lassen Sie nun die Ladungen der $ z $ -Achse wie zuvor mit einer Geschwindigkeit von $ + v $ bewegen. Bereits der Laborbeobachter UND DIE TESTLADUNG $ q $ sehen eine Kontraktion der Trennung gemäß $ Δz = Δz_0 \ sqrt {(1-v ^ 2 / c ^ 2)} $. Daher muss ein spezieller Verwandter nach denselben Manövern wie zuvor eine zusätzliche "Coulomb" -Abstoßung aufgrund der verdichteten Ladungsdichte vorhersagen. Die so vorhergesagte „magnetische“ Kraft muss also auf die RESTING-Ladung bei $ (1,0,0) $ einwirken. Und das wird nicht beobachtet. Nach meinem besten Wissen kann kein Strom entlang der $ z $ -Achse jemals eine Magnetkraft auf eine Ruheladung am Ursprung erzeugen.
  4. ol>

    Fazit: Im Gegensatz zu den Aussagen von White ist Magnetismus NICHT NUR Elektrostatik plus spezielle Relativitätstheorie. Eine solche reduktionistische Sichtweise wandelt Magnetismus in ein oberflächliches Spiel zwischen Referenzrahmen um

Nick Stauner: Danke für die Bearbeitung, aber die relativistischen Gleichungen sind alle falsch. Ich habe im Original hochgestellte Zeichen für v ^ 2 und c ^ 2 verwendet und auch für die Quadratwurzel, die ich verwendet habe ^ 1/2, aber jetzt erscheinen die Potenzen als tiefgestellte Zeichen und die Quadratwurzel wurde zum Bruchteil 1/2. Bitte beachten Sie, dass die Gleichungen wiederhergestellt sind. Vielen Dank
Vielen Dank, Kyle, für die Rekonstruktion meiner ursprünglichen Gleichungen
Soweit ich das beurteilen kann, ist dies keine gültige Kritik an der Antwort von White.Der einzig gültige Punkt ist 2., wo er tatsächlich einen numerischen Fehler gemacht hat, aber dies ändert nichts am qualitativen Ergebnis seiner Erklärung (und Ihre Version ist übrigens immer noch fehlerhaft).Alles, was Weiß sagt, ist, dass alle magnetischen Effekte, die in einem Trägheitsrahmen beobachtet werden, nur durch Coulomb-Kräfte im Restrahmen von $ q $ erklärt werden können, und ich sehe keinen Grund, warum Sie dem widersprechen.Insbesondere die "zusätzliche Kraft", die Sie in 3. beschreiben, die auf $ q $ einwirkt, ist keine zusätzliche Kraft im Restrahmen von $ q $.Nur wenn wir es verlassen.
Murod Abdukhakimov
2013-05-21 23:29:52 UTC
view on stackexchange narkive permalink

Ladung erzeugt ein Feld, das auf andere Ladungen einwirkt. Die Aktion dieses Feldes unterscheidet sich jedoch von verschiedenen Referenzrahmen.

Per Definition ist das elektrische Feld

  • etwas, das andere Ladungen beschleunigt und
  • Magnetfeld ist etwas, das andere Ladungen dreht.

Betrachten Sie Ladung in Ruhe. Es erzeugt nur ein elektrisches Feld in seinem Ruhezustand. In diesem Rahmen wirkt es auf andere Ladungen, indem es sie in Richtung des elektrischen Feldes $ \ textbf E $ beschleunigt. Was wir im Restrahmen der Ladung sehen, ist, dass die Impulsvektoren anderer Ladungen in diesem Rahmen "verstärkt" werden.

Wenn wir dies jedoch aus dem sich bewegenden Rahmen betrachten, werden wir das sehen Impulsvektoren anderer Ladungen sind nicht nur " beschleunigt ", sondern auch " gedreht ".

Dies liegt einfach an der "reinen" Beschleunigung in einem Rahmen sieht aus wie eine Kombination von Beschleunigung und Rotation in einem anderen Frame.

Um diesen "neuen Effekt" - die Rotation des Impulsvektors - zu berücksichtigen, sagen Physiker dies im zweiten Frame (d. h Bewegung der Ladung) gibt es ein Magnetfeld (zusätzlich zu dem elektrischen Feld, das (per Definition, siehe oben) nur andere Ladungen beschleunigt).

-1 Ein Magnetfeld beschleunigt auch Ladungen.
Rotation ist Beschleunigung, nicht wahr?
Ja, das ist richtig
Guill
2016-03-10 06:26:50 UTC
view on stackexchange narkive permalink

Eine einfache "erste Antwort" wäre die Verwendung der Analogie eines Bootes in einem See.Wenn sich das Boot auf der Wasseroberfläche bewegt, stört es das Wasser und erzeugt Wellen.Wenn es sich nicht bewegt, bewegt es sich nicht.

Wenn sich ein geladenes Teilchen durch das "durchdringende" EM-Feld (Raum) bewegt, stört es das EM-Feld und erzeugt ein Magnetfeld senkrecht zur Bewegungsrichtung des Partikels.

Anschließend können Sie eine oder alle anderen Antworten verwenden, die Sie erhalten haben, um weitere Details zu erfahren.

Hervorragende Antwort.Das $ \ mathbf {E} $ - Feld kann als eine Art "Flüssigkeit" betrachtet werden, die den Raum ausfüllt.Ein geladenes Teilchen, das sich durch diese Flüssigkeit bewegt, erzeugt senkrechte Wellen, die als $ \ mathbf {B} $ - Feld interpretiert werden können.Sie sind miteinander verbunden: Sie können nicht eins ohne das andere haben.
@Guill Entschuldigung für die dumme Frage, aber ist das Magnetfeld einfach die Bewegung / Welle (verursacht durch die Bewegung der Ladung) im Raum (die aus dem elektrischen Feld besteht, das wie Flüssigkeit ist)?
Amey Joshi
2013-05-21 16:06:41 UTC
view on stackexchange narkive permalink

Vielleicht möchten Sie sagen: "Das elektrische Feld einer Ladung in Ruhe erscheint als elektrisches Feld und als magnetisches Feld, wenn es von einem sich bewegenden Referenzrahmen aus betrachtet wird." Die Kommentare machen es richtig, eine Ladung ist mit einem elektromagnetischen Feld verbunden. Es erscheint als elektrostatisches Feld, wenn es von einem Rahmen aus betrachtet wird, in dem die Ladung ruht.

R. Romero
2019-02-27 02:17:16 UTC
view on stackexchange narkive permalink

Angenommen, Sie haben zwei Gebühren. Einer ist der Ursprung unseres Koordinatensystems. Die andere befindet sich an einer beliebigen Position $ (x, y, z) $ span> und nimmt an, dass eine magische Kraft sie dort hält, unabhängig davon, welche EM-Felder dort auftreten könnten.

Angenommen, die Ladung am Ursprung bewegt sich mit konstanter Geschwindigkeit in einer geraden Linie. Die Zielladung erhält nur mit Lichtgeschwindigkeit Aktualisierungen des Standorts der sich bewegenden Ladung. It reagiert auf die sich bewegende Ladung, nicht danach, wo sie sich jetzt befindet, sondern danach, wo sie einige Zeit in der Vergangenheit war.

Wenn sich die sich bewegende Ladung der Zielladung nähert, hebt ein Teil des Effekts den Effekt aufgrund der Ladung früher in ihrer Flugbahn auf. Das Gegenteil passiert, wenn sich die Ladung entfernt. Aufgrund der Überlappungseffekte kommt es zu einer gewissen Aufhebung des Feldes, wobei diese Aufhebung für die Komponente parallel zur Bewegungsrichtung erfolgt.

Eine sich bewegende Ladung trifft im Laufe der Zeit aus einer anderen Entfernung auf ein Ziel. Eine sich bewegende Ladung trifft im Laufe der Zeit aus einer anderen Richtung auf ein Ziel. Die sich ändernden Effekte verzögern sich, bevor sie das Ziel erreichen.

Ich mag Ihre Erklärung, da sie keine zwei getrennten Koordinatensysteme erfordert.Gibt es detaillierte Erklärungen, wie sich die Interaktion zwischen Ladungen ändert, wenn sich eine Ladung bewegt?Es wäre schön, eine diskrete Annäherung zu haben.
@user1700890: https://en.wikipedia.org/wiki/Li%C3%A9nard%E2%80%93Wiechert_potential Es ist ein elektrisches Potential für eine sich bewegende Ladung, das relativistische Effekte berücksichtigt.Das magnetische Potential wird genommen, indem es mit der Geschwindigkeit multipliziert und dann durch das Quadrat der Lichtgeschwindigkeit dividiert wird.Es gibt verschiedene "Korrekturfaktoren", die in den Ausdrücken für die Felder erscheinen und sich vom statischen Fall unterscheiden.


Diese Fragen und Antworten wurden automatisch aus der englischen Sprache übersetzt.Der ursprüngliche Inhalt ist auf stackexchange verfügbar. Wir danken ihm für die cc by-sa 3.0-Lizenz, unter der er vertrieben wird.
Loading...